Q19

 
walkerdoreen07
Thanks Received: 0
Forum Guests
 
Posts: 17
Joined: February 17th, 2010
 
 
trophy
First Responder
 

PT 30 S1 G4; Q19; If K is assembled on line 5

by walkerdoreen07 Mon Aug 02, 2010 6:35 pm

Please explain Q19 Why is not E?? Thank you!
User avatar
 
bbirdwell
Thanks Received: 864
Atticus Finch
Atticus Finch
 
Posts: 803
Joined: April 16th, 2009
 
 
 

Re: PT 30 S1 G4; Q19; If K is assembled on line 5

by bbirdwell Wed Aug 04, 2010 9:50 am

The question is asking for a pair of trucks that could appear consecutively.

If K is 5th, and we already know that S is 4th, our number line looks like this:

__ __ __ S K __ __

We know from the constraints that M must appear right before G:
MG. ("M is on the line numbered one lower than G...")

Thus, M could never appear right next to S because if M was third, G would have to be fourth, and this isn't possible. Likewise, M can't be fifth because K is.
I host free online workshop/Q&A sessions called Zen and the Art of LSAT. You can find upcoming dates here: http://www.manhattanlsat.com/zen-and-the-art.cfm
 
jeffshin720
Thanks Received: 0
Vinny Gambini
Vinny Gambini
 
Posts: 2
Joined: March 15th, 2012
 
 
 

Re: Q19

by jeffshin720 Tue Jun 26, 2012 8:32 pm

I'm having a hard time trying to figure out what this question is asking of me.

I made the number lines, but still confused with this question.
.
 
rpcuhk
Thanks Received: 5
Forum Guests
 
Posts: 41
Joined: May 02nd, 2011
 
 
 

Re: Q19

by rpcuhk Thu Jul 19, 2012 5:11 am

@jeffshin720

This question asks which answer choice contains two models that are next to each others
User avatar
 
ohthatpatrick
Thanks Received: 3807
Atticus Finch
Atticus Finch
 
Posts: 4661
Joined: April 01st, 2011
 
 
 

Re: Q19

by ohthatpatrick Sat Jul 21, 2012 2:42 pm

Given the condition that K is 5, and the original rules of the game, which answer choice shows two letters that could be right next to each other (in either order)?

We would start by plugging K=5 into our number line.

_ _ _ S K _ _

We always have S in spot 4, (rule 4).

Our other rules are
F - J
MG
H = 1 or 7

Since K doesn't itself trigger any rules, we should next consider our MG chunk (chunk rules are by far the most helpful / most limited parts of numbered ordering games).

In this case, it looks like MG could go 1/2, 2/3, or 6/7. We could either sketch out those 3 options or we could try to just approach the answer choices without sketching those out and mentally picture were the remaining pieces could / couldn't go.

People vary in their ability to mentally move around the other pieces, so let's go with the sketching out MG's three possibilities strategy:

M G _ S K _ _
_ M G S K _ _
_ _ _ S K M G

In all three scenarios, we still have F-J and H (1 or 7). For the top and bottom scenario, H's spot is determined.

M G _ S K _ H
_ M G S K _ _
H _ _ S K M G

Once we fill in H, we know we have to put F and J into the final two blanks in that order.

M G F S K J H
_ M G S K _ _
H F J S K M G

The middle scenario we should probably just leave as is, knowing we still have F-J and H (1 or 7). There are only 2 ways to complete the scenario, but we should be able to visualize those without having to write them in.

A) can G and H ever be next to each other? Nope, none of these scenarios would make that possible.

B) G and J are always separated.

C) H and J are next to each other in the top scenario. Done!

D) fyi, J and M can't touch. The only tempting possibility would be the 2nd scenario, but we know that J can't go in spot 1 (F-J).

E) we can see that M next to S never happens

Hope this helps.